0
$\begingroup$

Suppose $|f^\prime| \leq |g^\prime|$ on R and $f(0) = g(0) = 0$. Must $|f| \leq |g|$ on R?

[If we add the conditions that $g^\prime$ is either non-negative or non-positive on the half-line R+ and likewise on R-, then the result is an easy consequence of the Mean Value Thm. It seems as if it should be true as stated, but I don't see how to prove it.]

$\endgroup$
8
  • $\begingroup$ I think this is only true if f'(x), g'(x) have the same sign for all x, or at least a.e., You can have, e.g., f'(x)=1 , g'(x)=-1 in [0,1], while f'(x)=-1 and g'(x)=1 in $[1, \infty)$ $\endgroup$
    – gary
    Jul 15, 2017 at 1:03
  • 1
    $\begingroup$ Let $g(x) = 4x(1-x)$ for all $x \in \mathbb{R}$ and $f(x) = \begin{cases} 4x(1-x) & x \leq \frac 12 \\ 1 & x>\frac 12 \end{cases}$. We can see that $f(0) = g(0) = 0$, $|f'| \leq |g'|$ but $|f| > |g|$ when $x \in (\frac 12, 1)$. $\endgroup$ Jul 15, 2017 at 1:04
  • $\begingroup$ @gary, There's no differentiable $g$ satisfying your conditions. On the other hand, it is absolutely clear that functions differentiable almost everywhere have zero chance of satisfying the statement of the conjecture. $\endgroup$
    – xyzzyz
    Jul 15, 2017 at 1:13
  • $\begingroup$ @xyzzyz : Not so clear. Why can't you smooth out the functions at 1; in general, to achieve the main results? $\endgroup$
    – gary
    Jul 15, 2017 at 1:15
  • $\begingroup$ @xyzzyz: In my example, f.g are differentiable for $x \neq 1 $, i.e., a.e. $\endgroup$
    – gary
    Jul 15, 2017 at 1:23

3 Answers 3

2
$\begingroup$

Without the condition of non-changing sign of $g'$, $\lvert f\rvert$ can be larger than $\lvert g\rvert$. Consider for example

$$f(x) = \int_0^x \sin^2 t\,dt = \frac{1}{2} x - \frac{1}{4}\sin (2x)$$

and

$$g(x) = \int_0^x \sin t\,dt = 1 - \cos x.$$

Clearly $\lvert f'(x)\rvert = \sin^2 x \leqslant \lvert \sin x\rvert = \lvert g'(x)\rvert$ for all $x$, yet $g$ is bounded while $f$ is unbounded.

$\endgroup$
1
$\begingroup$

The assumption $$|f'(x)|\leq |g'(x)|\qquad\forall x\in{\mathbb R}\tag{1}$$ involves one free variable. One hopes to obtain a general statement involving two free variables. This is similar to the fact that $$|f'(x)|\leq M\qquad\forall x\in{\mathbb R}$$ implies $$|f(y)-f(x)|\leq M\>|y-x|\qquad\forall x, \ \forall y\ .$$ The best you can obtain from $(1)$ is the following: $${\rm Var}_J(f)\leq {\rm Var}_J(g)\qquad\forall\, J\ ,$$ whereby ${\rm Var}_J(f)$ denotes the total variation of $f$ on the interval $J=[x,y]$, $\>x<y$. A fortiori $$|f(y)-f(x)|\leq{\rm Var}_{[x,y]}(g)\ .$$

$\endgroup$
0
$\begingroup$

This question now seems to me cute but dumb.

Let $g(x) = 2x - x^2$. Let $f = g$ for $x \leq 1$ and $f = 2 - g$ for $x > 1$. Then $|f^\prime| = |g^\prime|, f(0) = g(0) = 0$, but $|f| > |g|$ for $x > 1$.

$\endgroup$

You must log in to answer this question.

Not the answer you're looking for? Browse other questions tagged .